Sie sind auf Seite 1von 70

Probability and Combinations Questions with Explanations

1. Mary and Joe are to throw three dice each. The score is the sum of points on all three
dice. If Mary scores 10 in her attempt what is the probability that Joe will outscore Mary in
his?
A. 24/64
B. 32/64
C. 36/64
D. 40/64
E. 42/64
Expected value of a roll of one dice is 1/6(1+2+3+4+5+6)=3.5.
Expected value of three dices is 3*3.5=10.5.
Mary scored 10 so the probability to have more then 10, or more than average is the same as to
have less than average=1/2.
P=1/2.
Answer: B.
Discussed at: http://gmatclub.com/forum/mother-mary-comes-to-me-86407.html
2. Denise is trying to open a safe whose combination she does not know. IF the safe has 4000
possible combinations, and she can try 75 different possibilities, what is the probability that
she does not pick the one correct combination.
A. 1
B. 159/160
C. 157/160
D. 3/160
E. 0
When trying the first time the probability Denise doesn't pick the correct
combination=3999/4000
Second time, as the total number of possible combinations reduced by one, not picking the right
one would be 3998/3999.
Third time 3997/3998
...
And the same 75 times.
So we get:

every denominator but the first will

cancel out and every nominator but the last will cancel out as well.
We'll get 3925/4000=157/160.
Answer: C.
Discussed at: http://gmatclub.com/forum/4000-possible-combination-84435.html

3. A box contains 10 pairs of shoes (20 shoes in total). If two shoes are selected at random,
what it is the probability that they are matching shoes?
A. 1/190
B. 1/20
C. 1/19
D. 1/10
E. 1/9
The probability would simply be: 1/1*1/19(as after taking one at random there are 19 shoes left
and only one is the pair of the first one) =1/19
Answer: C.
We can solve it in another way:
P= Favourable outcomes/Total # of outcomes
Favourable outcomes = 10C1 as there are 10 pairs and we need ONE from these 10 pairs.
Total # of outcomes= 20C2 as there are 20 shoes and we are taking 2 from them.
P=10C1/20C2 =10/(19*10)=1/19
Answer: C.
Discussed at: http://gmatclub.com/forum/probability-that-they-are-matching-shoes-85916.html
4. A Coach is filling out the starting lineup for his indoor soccer team. There are 10 boys on
the team, and he must assign 6 starters to the following positions: 1 goalkeeper, 2 on
defence, 2 in midfield, and 1 forward. Only 2 of the boys can play goalkeeper, and they
cannot play any other positions. The other boys can each play any of the other positions.
How many different groupings are possible?
A. 60
B. 210
C. 2580
D. 3360
E. 151200
2C1 select 1 goalkeeper from 2 boys;
8C2 select 2 defence from 8 boys (as 2 boys can only play goalkeeper 10-2=8);
6C2 select 2 midfield from 6 boys (as 2 boys can only play goalkeeper and 2 we've already
selected for defence 10-2-2=6);
4C1 select 1 forward from 4 boys (again as 2 boys can play only goalkeeper, 4 we've already
selected for defence and midfield 10-2-4=4)
Total # of selection=2C1*8C2*6C2*4C1=3360
Answer: D.
Discussed at: http://gmatclub.com/forum/combination-or-permutation-can-t-make-up-my-mind85800.html

5. In how many ways 8 different tickets can be distributed between Jane and Bill if each is
to receive any even number of tickets and all 8 tickets to be distributed.
A. From 2 to 6 inclusive.
B. From 98 to 102 inclusive.
C. From 122 to 126 inclusive.
D. From 128 to 132 inclusive.
E. From 196 to 200 inclusive.
Tickets can be distributed in the following ways:
{8,0} - 8C8=1
{6,2} - 8C6*2C2=28
{4,4} - 8C4*4C4=70
{2,6} - 8C2*6C6=28
{0,8} - 8C8=1
Total # of ways=1+28+70+28+1=128
Answer: D.
Discussed at: http://gmatclub.com/forum/sharing-tickets-87128.html
6. An insect has one shoe and one sock for each of its twelve legs. In how many different
orders can the insect put on its socks and shoes, assuming that, on each leg, the sock must
be put on before the shoe?
A.

B.

C.

D.
E.

Nothing like this will ever occur at real test, as this question is beyond the scope of GMAT. So
this question is just for practice.
NOTE that each sock and shoe is "assigned" to a specific leg.
Imagine situation with no restriction, meaning no need to put the socks before the shoes. In this
case the # of ways insect can put 24 items would be 24!. As we can choose to put ANY of 24
items first, then 23 items left, then 22 and so on.

Next step. On EACH leg we can put either sock OR shoe first. But for EACH leg from 12, only
one order is correct WITH restriction: sock first then shoe. For one leg chances of correct order
is 1/2, for two legs 1/2^2, similarly for 12 legs chances of correct order would 1/2^12.
So we get that for the total # of ways, WITH NO RESTRICTION, which is 24!, only 1/2^12 is
good WITH RESTRICTION.
So the final answer is 24!/2^12.
Answer: C.
Discussed at: http://gmatclub.com/forum/insect-87503.html
7. A man is known to speak truth 3 out of 4 times. He throws a die and reports that it is a
six. The probability that it is actually six is
A. 1/8
B. 2/8
C. 3/8
D. 1/2
P=Favorable outcomes/Total # of possible outcomes.
Favorable outcome is that it's actually six and he's telling the truth

Total # of possible outcomes is: either it's six and he's telling the truth OR it's not six and he's
telling the lie

Discussed at: http://gmatclub.com/forum/six-or-not-six-87984.html


8. In how many ways can 11 books on English and 9 books on French be placed in a row on
a shelf so that two books on French may not be together?
We have 11 English and 9 French books, no French books should be adjacent.
Imagine 11 English books in a row and empty slots like below:
*E*E*E*E*E*E*E*E*E*E*E*
Now if 9 French books would be placed in 12 empty slots, all French books will be separated by
English books.

So we can "choose" 9 empty slots from 12 available for French books, which is 12C9=220.
Answer: 220.
Discussed at: http://gmatclub.com/forum/permutation-combination-bookshelf-87352.html
9. In the xy-plane, the vertex of a square are (1, 1), (1,-1), (-1, -1), and (-1,1). If a point falls
into the square region, what is the probability that the ordinates of the point (x,y) satisfy
that x^2+y^2>1?
A. 1-pi/4
B. pi/2
C. 4-pi
D. 2-pi
E. pi-2
First note that the square we have is centered at the origin, has the length of the sides equal to 2
and the area equal to 4.
is an equation of a circle also centered at the origin, with radius 1 and
the

We are told that the point is IN the square and want to calculate the probability that it's outside
the circle (
means that the point is outside the given circle).

P=Favorable outcome/Total number of possible outcomes.


Favorable outcome is the area between the circle and the square=
Total number of possible outcomes is the area of the square (as given that the point is in the
square) =

Answer: A.
Discussed at: http://gmatclub.com/forum/probability-88246.html
10. How many words, with or without meaning can be made from the letters of the word
MONDAY, assuming that no letter is repeated, if 4 letters are used at a time?
A. 360
B. 720
C. 240

D. 120
E. 60
Choosing 4 letters out of 6 (distinct) letters to form the word = 6C4=15;
Permutations of these 4 letters = 4!=24;
Total # of words possible = 15*24= 360
Answer: A.
Discussed at: http://gmatclub.com/forum/permutation-question-88492.html
11. In how many ways can the letters of the word PERMUTATIONS be arranged if there
are always 4 letters between P and S
A. 2419200
B. 25401600
C. 1814400
D. 1926300
E. 1321500
There are 12 letters in the word "PERMUTATIONS", out of which T is repeated twice.
1. Choosing 4 letters out of 10 (12-2(P and S)=10) to place between P and S = 10C4 = 210;
2. Permutation of the letters P ans S (PXXXXS or SXXXXP) = 2! =2;
3. Permutation of the 4 letters between P and S = 4! =24;
4. Permutations of the 7 units {P(S)XXXXS(P)}{X}{X}{X}{X}{X}{X} = 7! = 5040;
5. We should divide multiplication of the above 4 numbers by 2! as there is repeated T.
Hence:

Answer: B.
Discussed at: http://gmatclub.com/forum/permutation-question-88492.html
12. 4 dices are thrown at the same time. whats the probability of getting ONLY 2 dices
showing the same face?
I suppose "only 2 dice showing the same face" means EXACTLY two? If so then:
Total # of outcomes = 6^4
Favorable outcomes = 4C2=6, choosing two dice which will provide the same face, these two
dice can take 6 values, other two 5 and 4. So, favorable outcomes=4C2*6*5*4.

Answer: 5/9.

Discussed at: http://gmatclub.com/forum/really-tough-88487.html


13. Right triangle ABC is to be drawn in the xy-plane so that the right angle is at A and AB
is parallel to the y-axis. If the x- and y-coordinates of A, B, and C are to be integers that are
consistent with the inequalities -6 x 2 and 4 y 9 , then how many different triangles
can be drawn that will meet these conditions?
A. 54
B. 432
C. 2160
D. 2916
E. 148,824
We have the rectangle with dimensions 9*6 (9 horizontal dots and 6 vertical). AB is parallel to yaxis and AC is parallel to x-axis.
Choose the (x,y) coordinates for vertex A: 9C1*6C1;
Choose the x coordinate for vertex C (as y coordinate is fixed by A): 8C1, (9-1=8 as 1 horizontal
dot is already occupied by A);
Choose the y coordinate for vertex B (as x coordinate is fixed by A): 5C1, (6-1=5 as 1 vertical
dot is already occupied by A).
9C1*6C*8C1*5C1=2160.
Answer: C.
Discussed at: http://gmatclub.com/forum/tough-problem-88958.html
14. In a certain game, a large bag is filled with blue, green, purple and red chips worth 1, 5,
x and 11 points each, respectively. The purple chips are worth more than the green chips,
but less than the red chips. A certain number of chips are then selected from the bag. If the
product of the point values of the selected chips is 88,000, how many purple chips were
selected?
A. 1
B. 2
C. 3
D. 4
E. 5
, as no chip's value is multiple of 2, hence 2^6=64 must be the product of
the values of the purple chips drawn. The value of the purple chip is multiple of 2, but more than
5 and less than 11, hence it's 8 (2^3). 8*8=64, two purple chips were drawn.
Answer: B (2).
Discussed at: http://gmatclub.com/forum/chips-worth-points-89694.html

15. In how many different ways can trhee letters be posted from seven different postboxes
assuming no two letters can be posted from the same postbox?
First letter could be sent from ANY of the seven postboxes - 7 (7 options);
Second letter could be sent from the SIX postboxes left - 6 (6 options);
Third letter could be sent from the FIVE postboxes left - 5 (5 options);
Total # of ways =7*6*5=210
What if there is no restriction, that is, if two or more letters can be posted from the same
box?
In this problem we don't have restriction, thus ANY letter could be sent from ANY postboxes
=7*7*7=7^3=343
Discussed at: http://gmatclub.com/forum/counting-principles-55340.html
16. Possible arrangements for the word REVIEW if one E can't be next to the other.
THEORY:
Permutations of n things of which P1 are alike of one kind, P2 are alike of second kind, P3 are
alike of third kind ...................... Pr are alike of r th kind such that: P1+P2+P3+..+Pr=n is:
.

For example number of permutation of the letters of the word "gmatclub" is 8! as there are 8
DISTINCT letters in this word.
Number of permutation of the letters of the word "google" is 6!/2!2!, as there are 6 letters out of
which "g" and "o" are represented twice.
Number of permutation of 9 balls out of which 4 are red, 3 green and 2 blue, would be 9!/4!3!2!.
In the original question there are 6 letters out of which E appears twice. Total number of
permutation of these letters (without restriction) would be:
.

# of combination for which two E are adjacent is

, (consider two E as one element

like: {R}{EE}{V}{I}{W}: # of permutation of this 5 elements is


Total # of permutation for which two E are not adjacent would be

)
.

Discussed at: http://gmatclub.com/forum/premutations-and-combinations-90174.html

17. In how many different ways can the letters A,A,B,B,B,C,D,E be arranged if the letter C
must be to the right of the letter D?
A. 1680
B. 2160
C. 2520
D. 3240
E. 3360
We have 8 letters out of which A appears twice and B appears three time. Total number of
permutation of these letters (without restriction) would be:
.

Now, in half of these cases D will be to the right of C and in half of these cases to the left, hence
the final answer would be

Answer: A.
Discussed at: http://gmatclub.com/forum/probability-q-91460.html
18. If there are 85 students in a statistics class and we assume that there are 365 days in a
year, what is the probability that at least two students in the class have the same birthday
(assuming birthdays are distributed independently)?
A. (85/365)* (84/364)
B. (1/365)* (1/364)
C. 1- (85!/365!)
D. 1- (365!/ 280! (365^85))
E. 1- (85!/(365^85))
The easiest way to solve this problem is to calculate opposite probability and subtract this value
from 1:
The opposite probability is that all students have the birthdays on different
days:
total 85 birthdays (first
student can have birthday on any day =1=365/365, the probability that the second student will
have the birthday on another day is 364/365, the probability that the third student will have the
birthday not on this two days is 363/365, and so on).
So, the probability that at least two students in the class have the same birthday
is:
.

Answer: D.

Discussed at: http://gmatclub.com/forum/probability-700-difficulty-level-92013.html


19. How many words can be formed by taking 4 letters at a time out of the letters of the
word MATHEMATICS.
There are 8 distinct letters: M-A-T-H-E-I-C-S. 3 letters M, A, and T are represented twice
(double letter).
Selected 4 letters can have following 3 patterns:
1. abcd - all 4 letters are different:
(choosing 4 distinct letters out of 8, when order matters) or
(choosing 4 distinct letters out of 8 when order does not matter and multiplying by 4! to get
different arrangement of these 4 distinct letters);
2. aabb - from 4 letters 2 are the same and other 2 are also the same:
- 3C2 choosing which two double letter will provide two letters (out of 3 double
letter - MAT), multiplying by
arranged in

to get different arrangements (for example MMAA can be

# of ways);

3. aabc - from 4 letters 2 are the same and other 2 are different:
- 3C1 choosing which letter will proved with 2 letters (out of 3 double
letter - MAT), 7C2 choosing third and fourth letters out of 7 distinct letters left and multiplying
by to get different arrangements (for example MMIC can be arranged in
# of ways).

1680+18+756=2454
Answer: 2454.
Discussed at: http://gmatclub.com/forum/tough-p-n-c-92675.html
20. In how many different ways can 4 physics, 2 math and 3 chemistry books be arranged
in a row so that all books of the same branch are together?
A. 1242
B. 1728
C. 1484
D. 1734
E. 1726
There are three branches, three units of books: {physics}{math}{chemistry} - aranging branches

3!;
Arranging the books within the branches:
physics - 4!;
math - 2!;
chemistry - 3!;
Total: 3!*4!*2!*3!.
Answer: B.
Discussed at: http://gmatclub.com/forum/perm-55383.html
21. In how many ways can the letters of the word PERMUTATIONS be arranged if there
are always 4 letters between P and S?
There are 12 letters in the word "PERMUTATIONS", out of which T is repeated twice.
1. Choosing 4 letters out of 10 (12-2(P and S)=10) to place between P and S = 10C4 = 210;
2. Permutation of the letters P ans S (PXXXXS or SXXXXP) = 2! =2;
3. Permutation of the 4 letters between P and S = 4! =24;
4. Permutations of the 7 units {P(S)XXXXS(P)}{X}{X}{X}{X}{X}{X} = 7! = 5040;
5. We should divide multiplication of the above 4 numbers by 2! as there is repeated T.
Hence:

Discussed at: http://gmatclub.com/forum/interesting-problems-of-permutations-andcombinations-94381.html


22. In how many of the distinct permutations of the letters in the word MISSISSIPPI do the
4 I's not come together?
There are 11 letters in the word "MISSISSIPPI ", out of which: M=1, I=4, S=4, P=2.
Total # of permutations is

# of permutations with 4 I's together is

. Consider 4 I's as one unit: {M}{S}{S}{S}{S}{P}

{P}{IIII} - total 8 units, out of which {M}=1, {S}=4, {P}=2, {IIII}=1.


So # of permutations with 4 I's not come together is:

Discussed at: http://gmatclub.com/forum/interesting-problems-of-permutations-andcombinations-94381.html

22. A firm is divided into four departments, each of which contains four people. If a
project is to be assigned to a team of three people, none of which can be from the same
department, what is the greatest number of distinct teams to which the project could be
assigned?
A 4^3
B. 4^4
C. 4^5
D. 6(4^4)
E. 4(3^6)
# of ways to choose which 3 department will provide employee for the team and as each
chosen department can provide with 4 employees then total # of different teams will
be
.

Discussed at: http://gmatclub.com/forum/combinations-problem-extreme-challenge-a-firm-is84160.html


23. How many positive integers less than 10,000 are there in which the sum of the digits
equals 5?
A. 31
B. 51
C. 56
D. 62
E. 93
Consider this: we have 5 's and 3 separators , like:

. How many permutations

(arrangements) of these symbols are possible? Total of 8 symbols (5+3=8), out of which 5 's
and 3 's are identical, so

With these permutations we'll get combinations like:


OR

this would be 3 digit number 212

this would be single digit number 5 (smallest number less than 10,000 in which

sum of digits equals 5) OR

this would be 4 digit number 5,000 (largest number less

than 10,000 in which sum of digits equals 5)...


Basically this arrangements will give us all numbers less than 10,000 in which sum of the digits
(sum of 5 d's=5) equals 5.

Hence the answer is

Answer: C (56).
This can be done with direct formula as well:
The total number of ways of dividing n identical items (5 d's in our case) among r persons or
objects (4 digt places in our case), each one of whom, can receive0, 1, 2 or more items (from
zero to 5 in our case) is
.

In our case we'll get:

Discussed at: http://gmatclub.com/forum/integers-less-than-85291.html


24. A local bank that has 15 branches uses a two-digit code to represent each of its
branches. The same integer can be used for both digits of a code, and a pair of two-digit
numbers that are the reverse of each other (such as 17 and 71) are considered as two
separate codes. What is the fewest number of different integers required for the 15 codes?
A. 3
B. 4
C. 5
D. 6
E. 7
Consider the code XY. If there are
take

values, thus from

digits available then X can take

digits we can form

values and Y can also

different 2-digit codes: this is the same as

from 10 digits (0, 1, 2, 3, ..., 9) we can form 10^2=100 different 2-digit numbers (00, 01, 02, ...,
99).
We want # of codes possible from

digit to be at least 15 -->

-->

, hence min 4

digits are required.


Answer: B.
Discussed at: http://gmatclub.com/forum/permutation-question-98109.html
25. Mrs. Smith has been given film vouchers. Each voucher allows the holder to see a film
without charge. She decides to distribute them among her four nephews so that each
nephew gets at least two vouchers. How many vouchers has Mrs. Smith been given if there
are 120 ways that she could distribute the vouchers?
A. 13
B. 14

C. 15
D. 16
E. more than 16
Clearly there are more than 8 vouchers as each of four can get at least 2. So, basically 120 ways
vouchers can the distributed are the ways to distribute
vouchers, so that each can get from
zero to

as at "least 2", or 2*4=8, we already booked. Let

In how many ways we can distribute

be

identical things among 4 persons? Well there is a

formula for this but it's better to understand the concept.


Let

. And imagine we want to distribute 5 vouchers among 4 persons and each can get

from zero to 5, (no restrictions).


Consider:

We have 5 tickets (t) and 3 separators between them, to indicate who will get the tickets:

Means that first nephew will get all the tickets,

Means that first got 0, second 1, third 3, and fourth 1


And so on.
How many permutations (arrangements) of these symbols are possible? Total of 8 symbols
(5+3=8), out of which 5 's and 3 's are identical, so
. Basically it's the number of
ways we can pick 3 separators out of 5+3=8:

So, # of ways to distribute 5 tickets among 4 people is

For

it will be the same: # of ways to distribute

get from zero to

) would be

tickets among 4 persons (so that each can


.

. -->
earlier:

. Plus the 8 tickets we booked

Answer: C (15).
P.S. How to solve

: 720, three digit integer ending with 0,

is the product of three consecutive integers. Obviously one of them must be multiple of 5:
try
, next possible triplet
, OK. So
-->

P.P.S. Direct formula:


The total number of ways of dividing n identical items among r persons, each one of whom,
can receive 0,1,2 or more items is
.

The total number of ways of dividing n identical items among r persons, each one of whom
receives at least one item is
.

Discussed at: http://gmatclub.com/forum/voucher-98225.html


26. In the Mundane Goblet competition, 6 teams compete in a round robin format: that
is, each team plays every other team exactly once. A team gets 3 points for a win, 1 point for
a tie (a draw), and 0 points for a loss. What is the difference between the maximum total
points and the minimum total points that can be gained by all teams (added together) in the
Mundane Goblet competition?
A. 15
B. 30
C. 45
D. 60
E. 75
There will be

games needed so that each team to play every other team exactly once (

is the # of ways we can pick two different teams to play each other).

Now, in one game max points (3 points) will be obtained if one team wins and another looses
and min points (1+1=2 points) will be obtained if there will be a tie. Hence, maximum points that
can be gained by all teams will be 15 games * 3 points=45 and the minimum points that can be
gained by all teams will be 15 games * 2 points=30, difference is 45-30=15.
Answer: A.

Discussed at: http://gmatclub.com/forum/mbamission-the-quest-for-96952.html


26. Seven men and five women have to sit around a circular table so that no 2 women are
together. In how many different ways can this be done?
# of arrangements of 7 men around a table is

There will be 7 possible places for women between them, 7 empty slots. # of ways to choose in
which 5 slots women will be placed is
;
# of arrangements of 5 women in these slots is
So total:

Answer: 1,814,400.
Discussed at: http://gmatclub.com/forum/arrangement-in-a-circle-98185.html
27. If x is a randomly chosen integer between 1 and 20, inclusive, and y is a randomly
chosen integer between 21 and 40, inclusive, what is the probability that xy is a multiple of
4?
A. 1/4
B. 1/3
C. 3/8
D. 7/16
E.
Let's find the probability of an opposite event and subtract this value from 1.
There are three cases xy NOT to be a multiple of 4:
A. both x and y are odd --> 1/2*1/2=1/4;
B. x is odd and y is even but not multiple of 4 --> 1/2*1/4=1/8;
C. y is odd and x is even but not multiple of 4 --> 1/2*1/4=1/8.

Answer: E.
Discussed at: http://gmatclub.com/forum/hard-one-98842.html
28. A password to a certain database consists of digits that cannot be repeated.If the
password is known to consist of at least 8 digits and it takes 12 seconds to try one
combination , what is the amount of time , in minutes , necessary to guarantee access to
database?
A. 8!/5
B. 8!/2

C. 8!
D. 10!/2
E. 5/2.10!
Password can have 8, 9 or 10 digits (more than 10 is not possible as per stem digits must be
distinct).
Total # of passwords possible for 8 digits is

Total # of passwords possible for 9 digits is

Total # of passwords possible for 10 digits is

Time needed to guarantee access to database is

minutes.

Answer: D.
Discussed at: http://gmatclub.com/forum/permutation-and-combination-95496.html
29. A row of seats in a movie hall contains 10 seats. 3 Girls & 7 boys need to occupy those
seats. What is the probability that no two girls will sit together?
Consider the following:
*B*B*B*B*B*B*B*
Now, if girls will occupy the places of 8 stars no girl will sit together.
# of ways 3 girls can occupy the places of these 8 stars is
# of ways 3 girls can be arranged on these places is
# of ways 7 boys can be arranged is

So total # of ways to arrange 3 Girls and 7 boys so that no girls are together is
Total # of ways to arrange 10 children is
So

Discussed at: http://gmatclub.com/forum/ps-probability-3girls-7-boys-99268.html

30. A fair coin is tossed 5 times. What is the probability of getting at least three heads on
consecutive tosses?
A 2/16
B 1/4
C 7/24
D 5/16
E 15/32
At least 3 heads means 3, 4, or 5 heads.
3 consecutive heads
5 cases:
HHHTT
THHHT
TTHHH
HTHHH
HHHTH
.

4 consecutive heads
2 cases:
HHHHT
THHHH
.

5 consecutive heads
1 case:
HHHHH
.

Answer: B.
Discussed at: http://gmatclub.com/forum/hard-probability-99478.html
31. Bill has a small deck of 12 playing cards made up of only 2 suits of 6 cards each. Each of
the 6 cards within a suit has a different value from 1 to 6; thus, for each value from 1 to 6,
there are two cards in the deck with that value. Bill likes to play a game in which he

shuffles the deck, turns over 4 cards, and looks for pairs of cards that have the same value.
What is the chance that Bill finds at least one pair of cards that have the same value?
A. 8/33
B. 62/165
C. 17/33
D. 103/165
E. 25/33
Let's calculate the opposite probability ans subtract this value from 1.
Opposite probability would be that there will be no pair in 4 cards, meaning that all 4 cards will
be different:
.

- # of ways to choose 4 different cards out of 6 different values;


- as each of 4 cards chosen can be of 2 different suits;
- total # of ways to choose 4 cards out of 12.

So

Or another way:
We can choose any card for the first one -

Next card can be any card but 1 of the value we'v already chosen -

(if we've picked 3, then

there are one more 3 left and we can choose any but this one card out of 11 cards left);
Next card can be any card but 2 of the values we'v already chosen (if we've picked 3 and 5,
then there are one 3 and one 5 left and we can choose any but these 2 cards out of 10 cards left);
Last card can be any card but 3 of the value we'v already chosen - ;

So

- the same answer as above.

Answer: C.
Discussed at: http://gmatclub.com/forum/probability-96078.html
32. Six mobsters have arrived at the theater for the premiere of the film Goodbuddies.
One of the mobsters, Frankie, is an informer, and he's afraid that another member of his
crew, Joey, is on to him. Frankie, wanting to keep Joey in his sights, insists upon standing
behind Joey in line at the concession stand, though not necessarily right behind him. How
many ways can the six arrange themselves in line such that Frankies requirement is
satisfied?
A. 6
B. 24
C. 120
D. 360
E. 720
Arrangement of 6=6!. In half of the cases Frankie will be behind Joey and in half of the cases
Joey will be behind Frankie (as probability doesn't favor any of them). So, the needed
arrangement is 6!/2=360.
Answer: D (360)
Discussed at: http://gmatclub.com/forum/permutation-combination-86147.html
33. In a room filled with 7 people, 4 people have exactly 1 sibling in the room and 3 people
have exactly 2 siblings in the room. If two individuals are selected from the room at
random, what is the probability that those two individuals are NOT siblings?
A. 5/21
B. 3/7
C. 4/7
D. 5/7
E. 16/21
As there are 4 people with exactly 1 sibling each: we have two pairs of siblings (1-2; 3-4).
As there are 3 people with exactly 2 siblings each: we have one triple of siblings (4-5-6).
Solution #1:
# of selections of 2 out of 7 -

# of selections of 2 people which are not siblings from second pair of siblings)+
+

(one from first pair of siblings*one

(one from first pair of siblings*one from triple)

(one from second pair of siblings*one from triple)

Solution #2:
# of selections of 2 out of 7 -

# of selections of 2 siblings -

Solution #3:
.

Answer: E.
Discussed at: http://gmatclub.com/forum/mgmat-cat-picking-friends-87550.html
34. At a blind taste competition a contestant is offered 3 cups of each of the 3 samples of tea
in a random arrangement of 9 marked cups. If each contestant tastes 4 different cups of
tea, what is the probability that a contestant does not taste all of the samples?
A.
B.
C.
D.
E.

"The probability that a contestant does not taste all of the samples" means that contestant tastes
only 2 samples of tea (one sample is not possible as contestant tastes 4 cups>3 of each kind).
.

- # of ways to choose which 2 samples will be tasted;


- # of ways to choose 4 cups out of 6 cups of two samples (2 samples*3 cups each = 6 cups);
- total # of ways to choose 4 cups out of 9.

Answer: B.
Another way:
Calculate the probability of opposite event and subtract this value from 1.
Opposite event is that contestant will taste ALL 3 samples, so contestant should taste 2 cups of
one sample and 1 cup from each of 2 other samples (2-1-1).
- # of ways to choose the sample which will provide with 2 cups;
- # of ways to chose these 2 cups from the chosen sample;
- # of ways to chose 1 cup out of 3 from second sample;
- # of ways to chose 1 cup out of 3 from third sample;
- total # of ways to choose 4 cups out of 9.

Answer: B.
Discussed at: http://gmatclub.com/forum/prob-gclub-diagonostics-86830.html
35. A man chooses an outfit from 3 different shirts, 2 different pairs of shoes, and 3
different pants. If he randomly selects 1 shirt, 1 pair of shoes, and 1 pair of pants each
morning for 3 days, what is the probability that he wears the same pair of shoes each day,
but that no other piece of clothing is repeated?
A (1/3)^6 *(1/2)^3
B (1/3)^6*(1/2)
C (1/3)^4
D (1/3)^2*(1/2)
E 5(1/3)^2

For the first day he can choose any outfit,

For the second day he must choose the same shoes as on the first day and different shirts and
pants form the first day's,
;
For the third day he must choose the same shoes as on the first day and different shirts and pants
from the first and second day's,
;

Answer: C -

Discussed at: http://gmatclub.com/forum/probability-of-wearing-dress-91717.html


36. Anthony and Michael sit on the six-member board of directors for company X. If the
board is to be split up into 2 three-person subcommittees, what percent of all the possible
subcommittees that include Michael also include Anthony?
A. 20%
B. 30%
C. 40%
D. 50%
E. 60%
First approach:
Let's take the group with Michael: there is a place for two other members and one of them should
be taken by Anthony, as there are total of 5 people left, hence there is probability of 2/5=40%.
Second approach:
Again in Michael's group 2 places are left, # of selections of 2 out of 5 5C2=10 - total # of
outcomes.
Select Anthony - 1C1=1, select any third member out of 4 - 4C1=4, total # =1C1*4C1=4 - total #
of winning outcomes.
P=# of winning outcomes/# of outcomes=4/10=40%
Third approach:
Michael's group:
Select Anthony as a second member out of 5 - 1/5 and any other as a third one out of 4 left 4/4,
total=1/5*4/4=1/5;
Select any member but Anthony as second member out of 5 - 4/5 and Anthony as a third out of 4
left 1/4, total=4/5*1/4=1/5;
Sum=1/5+1/5=2/5=40%
Fourth approach:
Total # of splitting group of 6 into two groups of 3: 6C3*3C3/2!=10
# of groups with Michael and Anthony: 1C1*1C1*4C1=4

P=4/10=40%
Answer: C.
Discussed at: http://gmatclub.com/forum/combination-anthony-and-michael-sit-on-the-sixmember-87081.html
37. Two couples and one single person are seated at random in a row of five chairs. What is
the probability that neither of the couples sits together in adjacent chairs?
A 1/5
B. 1/4
C. 3/8
D. 2/5
E. 1/2
Let's find the opposite probability and subtract it from 1.
Opposite event that neither of the couples sits together is event that at leas one couple sits
together. # of arrangements when at leas one couple sits together is sum of arrangements when
EXACTLY 2 couples sit together and EXACTLY 1 couples sit together.
Couple A: A1, A2
Couple B: B1, B2
Single person: S
EXACTLY 2 couples sit together:
Consider each couple as one unit: {A1A2}{B1B2}{S}, # of arrangement would
be:
. 3! # of different arrangement of these 3 units, 2! arrangement of couple A
(A1A2 or A2A1), 2! arrangement of couple B (B1B2 or B2B1).
EXACTLY 1 couples sit together:
Couple A sits together: {A1A2}{B1}{B2}{S}, # of arrangement would be:

. 4! # of

different arrangement of these 4 units, 2! arrangement of couple A (A1A2 or A2A1). But these
48 arrangements will also include arrangements when 2 couples sit together, so total for couple A
would be
;
The same for couple B: {B1B2}{A1}{A2}{S}, # of arrangement would be:

. Again

these 48 arrangements will also include arrangements when 2 couples sit together, so total for
couple B would be
;
.

Finally we get the # of arrangements when at least one couple sits together is

.
Total # of arrangements of 5 people is
couple sits together would be

, hence probability of an event that at leas one


.

So probability of an event that neither of the couples sits together would be

Answer: D.
Discussed at: http://gmatclub.com/forum/2-couples-and-a-single-person-probability-question92400.html
38. As part of a game, four people each must secretly choose an integer between 1 and 4,
inclusive.
What is the approximate likelihood that 2 people will choose same number?
What is the approximate likelihood that 3 people will choose same number?
When four people choose an integer between 1 and 4, inclusive 5 cases are possible:
A. All choose different numbers - {a,b,c,d};
B. Exactly 2 people choose same number and other 2 choose different numbers - {a,a,b,c};
C. 2 people choose same number and other 2 also choose same number - {a,a,b,b};
D. 3 people choose same number - {a,a,a,b};
E. All choose same number - {a,a,a,a}.
Some notes before solving:
As only these 5 cases are possible then the sum of their individual probabilities must be
1:

As each person has 4 options, integers from 1 to 4, inclusive, thus denominator, total # of
outcomes would be 4^4 for all cases.
A. All choose different numbers - {a,b,c,d}:
.

# of ways to "assign" four different objects (numbers 1, 2, 3, and 4) to 4 persons is 4!.


B. Exactly 2 people choose same number and other 2 choose different numbers - {a,a,b,c}:

- # of ways to choose which 2 persons will have the same number;


- # of ways to choose which number it will be;
- # of ways to choose 2 different numbers out of 3 left for 2 other persons when order
matters;
C. 2 people choose same number and other 2 also choose same number - {a,a,b,b}:
.

- # of ways to choose which 2 numbers out of 4 will be used in {a,a,b,b};


- # of ways to "assign" 4 objects out of which 2 a's and 2 b's are identical to 4 persons;

D. 3 people choose same number - {a,a,a,b}:


.

- # of ways to choose which 3 persons out of 4 will have same number;


- # of ways to choose which number it will be;
- options for 4th person.
E. All choose same number - {a,a,a,a}:
.

- options for the number which will be the same.

Checking:

Discussed at: http://gmatclub.com/forum/probability-simple-question-98684.html


39. Diana is going on a school trip along with her two brothers, Bruce and Clerk. The
students are to be randomly assigned into 3 groups, with each group leaving at a different
time. What is the probability that DIana leaves at the same time as AT LEAST on her
bothers?
A. 1/27
B. 4/27
C. 5/27
D. 4/9
E. 5/9
Diana and her two brothers can be assigned to one of the 3 groups, so each has 3 choices, so total
# of different assignments of Diana and her 2 brothers to 3 groups is
.

Now, "Diana leaves at the same time as AT LEAST one her brothers" means that Diana is in the
same group as at least one her brothers.
Let's find the opposite probability of such event and subtract it from 1. Opposite probability
would be the probability that Diana is not in the group with any of her brothers.
In how many ways we can assign Diana and her two brothers to 3 groups so that Diana is not in
the group with any of her brothers? If Diana is in the first group, then each of her two brothers
will have 2 choices (either the second group or the third) and thus can be assigned
in
ways to other two groups. As there are 3 groups, then total # of ways to assign
Diana and her 2 brothers to these groups so that Diana is not in the group with any her bothers
is
(For each of Diana's choices her brother can be assigned in 4 ways, as Diana has 3
choices: first, second or the third group, then total

). So probability of this event is

Probability that Diana is in the same group as at least one her brothers would be

Answer: E.
Discussed at: http://gmatclub.com/forum/probability-qs-from-princeton-review-96989.html
40. Kate and David each have $10. Together they flip a coin 5 times. Every time the coin
lands on heads, Kate gives David $1. Every time the coin lands on tails, David gives Kate
$1. After the coin is flipped 5 times, what is the probability that Kate has more than $10 but

less than $15?


A. 5/16
B. 15/32
C. 1/2
D. 21/32
E. 11/16
After 5 tries Kate to have more than initial sum of 10$ and less than 15$ must win 3 or 4 times
(if she wins 2 or less times she'll have less than 10$ and if she wins 5 times she'll have 15$).
So the question becomes "what is the probability of getting 3 or 4 tails in 5 tries?".

Answer: B.
To elaborate more:
If the probability of a certain event is

, then the probability of it occurring

times in

-time

sequence is:

For example for the case of getting 3 tails in 5 tries:


(5 tries);
(we want 3 tail);
(probability of tail is 1/2).

So,

OR: probability of scenario t-t-t-h-h is

, but t-t-t-h-h can occur in different ways:

t-t-t-h-h - first three tails and fourth and fifth heads;


h-h-t-t-t - first two heads and last three tails;
t-h-h-t-t - first tail, then two heads, then two tails;
...
Certain # of combinations. How many combinations are there? Basically we are looking at # of
permutations of five letters t-t-t-h-h, which is
.

Hence

Discussed at: http://gmatclub.com/forum/probability-97177.html


1. Stanstard Deviation
1. A set of data consists of the following 5 numbers: 0,2,4,6, and 8. Which two
numbers, if added to create a set of 7 numbers, will result in a new standard
deviation that is close to the standard deviation for the original 5 numbers?
(A) -1 and 9
(B) 4 and 4
(C) 3 and 5
(D) 2 and 6
(E) 0 and 8
2. A certain list of 100 data has an average of 6 and standard deviation of d where d
is positive. Which of the following pairs of data, when added to the list must result in
a list of 102 data with the standard deviation less than d?
(A) 0 and 6
(B) 0 and 12
(C) 0 and 0
(D) -6 and 0
(E) 6 and 6
3. For a certain examination, a score of 58 was 2 standard deviations below the
mean, and a score of 98 was 3 standard deviations above the mean. What was the
mean score for the examination?
(A) 74
(B) 76
(C) 78
(D) 80
(E) 82
4. Which of the following distribution of numbers has the greatest standard
deviation?
(A) {-3, 1, 2}
(B) {-2, -1, 1, 2}
(C) {3, 5, 7}
(D) {-1, 2, 3, 4}
(E) {0, 2, 4}
5. Which of the following has the same standard deviation as {s,r,t}?
I. {r-2, s-2, t-2}
II. {0, s-t, s-r}
III. {|r|, |s|, |t|}
(A) I only
(B) II only

(C) III only


(D) I and II only
(E) I and III only
6. A certain characteristic in a large population has a distribution that is symmetric
about the mean m. If 68% of the distribution lies one standard deviation d of the
mean, what percent of the distribution is less than m+d?
(A) 16%
(B) 32%
(C) 48%
(D) 84%
(E) 92%
7. Which of the following data sets has the third largest standard deviation?
(A) {1, 2, 3, 4, 5}
(B) {2, 3, 3, 3, 4}
(C) {2, 2, 2, 4, 5}
(D) {0, 2, 3, 4, 6}
(E) {-1, 1, 3, 5, 7}
8. The table below represents three sets of numbers with their respective medians,
means and standard deviations. The third set, Set [A+B], denotes the set that is
formed by combining Set A and Set B.
Median Mean StandardDeviation
Set A: X, Y, Z.
Set B: L, M, N.
Set [A + B]: Q, R, S.
If X Y > 0 and L M = 0, then which of the following must be true?
I. Z > N
II. R > M
III. Q > R
(A) I only
(B) II only
(C) III only
(D) I and II only
(E) None
9. E is a collection of four odd integers and the greatest difference between any two
integers in E is 4. The standard deviation of E must be one of how many numbers?
(A) 3
(B) 4
(C) 5
(D) 6
(E) 7
10. If a certain sample of data has a mean of 20.0 and a standard deviation of 3.0,
which of the following values is more than 2.5 standard deviations from the mean?
(A) 12.0
(B) 13.5
(C) 17.0
(D) 23.5

(E) 26.5
11. Arithmetic mean and standard deviation of a certain normal distribution are
13.5 and 1.5. What value is exactly 2 standard deviations less than the mean?
(A) 10.5
(B) 11
(C) 11.5
(D) 12
(E) 12.5
CALCULATING STANDARD DEVIATION OF A SET {x1, x2, ... xn}:
1. Find the mean, m, of the values.
2. For each value xi calculate its deviation (xi-m) from the mean.
3. Calculate the squares of these deviations.
4. Find the mean of the squared deviations. This quantity is the variance.
5. Take the square root of the variance. The quantity is th SD.
TIPS:
1. |Median-Mean| <= SD.
2. Variance is the square of the standard deviation.
3. If Range or SD of a list is 0, then the list will contain all identical elements. And
vise versa: if a list contains all identical elements then the range and SD of a list is 0.
If the list contains 1 element: Range is zero and SD is zero.
4. SD is always >=0. SD is 0 only when the list contains all identical elements (or
which is same only 1 element).
5. Symmetric about the mean means that the shape of the distribution on the right
and left side of the curve are mirror-images of each other.
6. If we add or subtract a constant to each term in a set:
Mean will increase or decrease by the same constant.
SD will not change.
7. If we increase or decrease each term in a set by the same percent:
Mean will increase or decrease by the same percent.
SD will increase or decrease by the same percent.
8. Changing the signs of the element of a set (multiplying by -1) has no effect on SD.
9. The SD of any list is not dependent on the average, but on the deviation of the
numbers from the average. So just by knowing that two lists having different
averages doesn't say anything about their standard deviation - different averages
can have the same SD.
2.
1. A set of data consists of the following 5 numbers: 0,2,4,6, and 8. Which two
numbers, if added to create a set of 7 numbers, will result in a new standard
deviation that is close to the standard deviation for the original 5 numbers?

(A) -1 and 9
(B) 4 and 4
(C) 3 and 5
(D) 2 and 6
(E) 0 and 8
Answer: D.
2. A certain list of 100 data has an average of 6 and standard deviation of d where d
is positive. Which of the following pairs of data, when added to the list must result in
a list of 102 data with the standard deviation less than d?
(A) 0 and 6
(B) 0 and 12
(C) 0 and 0
(D) -6 and 0
(E) 6 and 6
Answer: E.
3. For a certain examination, a score of 58 was 2 standard deviations below the
mean, and a score of 98 was 3 standard deviations above the mean. What was the
mean score for the examination?
(A) 74
(B) 76
(C) 78
(D) 80
(E) 82
Answer: A.
4. Which of the following distribution of numbers has the greatest standard
deviation?
(A) {-3, 1, 2}
(B) {-2, -1, 1, 2}
(C) {3, 5, 7}
(D) {-1, 2, 3, 4}
(E) {0, 2, 4}
Answer: A.
5. Which of the following has the same standard deviation as {s,r,t}?
I. {r-2, s-2, t-2}
II. {0, s-t, s-r}
III. {|r|, |s|, |t|}
(A) I only
(B) II only
(C) III only
(D) I and II only
(E) I and III only
Answer: D.
6. A certain characteristic in a large population has a distribution that is symmetric
about the mean m. If 68% of the distribution lies one standard deviation d of the
mean, what percent of the distribution is less than m+d?
(A) 16%
(B) 32%

(C) 48%
(D) 84%
(E) 92%
Answer: D.
7. Which of the following data sets has the third largest standard deviation?
(A) {1, 2, 3, 4, 5}
(B) {2, 3, 3, 3, 4}
(C) {2, 2, 2, 4, 5}
(D) {0, 2, 3, 4, 6}
(E) {-1, 1, 3, 5, 7}
Answer: A.
8. The table below represents three sets of numbers with their respective medians,
means and standard deviations. The third set, Set [A+B], denotes the set that is
formed by combining Set A and Set B.
Median Mean StandardDeviation
Set A: X, Y, Z.
Set B: L, M, N.
Set [A + B]: Q, R, S.
If X Y > 0 and L M = 0, then which of the following must be true?
I. Z > N
II. R > M
III. Q > R
(A) I only
(B) II only
(C) III only
(D) I and II only
(E) None
Answer: E
9. E is a collection of four odd integers and the greatest difference between any two
integers in E is 4. The standard deviation of E must be one of how many numbers?
(A) 3
(B) 4
(C) 5
(D) 6
(E) 7
Answer: B
10. If a certain sample of data has a mean of 20.0 and a standard deviation of 3.0,
which of the following values is more than 2.5 standard deviations from the mean?
(A) 12.0
(B) 13.5
(C) 17.0
(D) 23.5
(E) 26.5
Answer: A.
11. Arithmetic mean and standard deviation of a certain normal distribution are
13.5 and 1.5. What value is exactly 2 standard deviations less than the mean?

(A) 10.5
(B) 11
(C) 11.5
(D) 12
(E) 12.5
Answer: A.
Tough and Tricky Problems
1. THE SUM OF EVEN INTEGERS:
The sum of the even numbers between 1 and k is 79*80, where k is an odd number, then k=?
(A) 79
(B) 80
(C) 81
(D) 157
(E) 159
2. THE PRICE OF BUSHEL:
The price of a bushel of corn is currently $3.20, and the price of a peck of wheat is $5.80. The
price of corn is increasing at a constant rate of 5x cents per day while the price of wheat is
decreasing at a constant rate of 2^1/2*x-x cents per day. What is the approximate price when a
bushel of corn costs the same amount as a peck of wheat?
(A) $4.50
(B) $5.10
(C) $5.30
(D) $5.50
(E) $5.60
3. LEAP YEAR:
How many randomly assembled people are needed to have a better than 50% probability that at
least 1 of them was born in a leap year?
A. 1
B. 2
C. 3
D. 4
E. 5
4. ADDITION PROBLEM:
AB + CD = AAA, where AB and CD are two-digit numbers and AAA is a three digit number; A,
B, C, and D are distinct positive integers. In the addition problem above, what is the value of C?
(A) 1
(B) 3
(C) 7
(D) 9
(E) Cannot be determined
5. RACE:
A and B ran, at their respective constant rates, a race of 480 m. In the first heat, A gives B a head

start of 48 m and beats him by 1/10th of a minute. In the second heat, A gives B a head start of
144 m and is beaten by 1/30th of a minute. What is Bs speed in m/s?
(A) 12
(B) 14
(C) 16
(D) 18
(E) 20
6. PROBABILITY OF DRAWING:
A bag contains 3 red, 4 black and 2 white balls. What is the probability of drawing a red and a
white ball in two successive draws, each ball being put back after it is drawn?
(A) 2/27
(B) 1/9
(C) 1/3
(D) 4/27
(E) 2/9
7. THE DISTANCE BETWEEN THE CIRCLE AND THE LINE:
What is the least possible distance between a point on the circle x^2 + y^2 = 1 and a point on the
line y = 3/4*x - 3?
A) 1.4
B) sqrt (2)
C) 1.7
D) sqrt (3)
E) 2.0
8. THE AVERAGE TEMPERATURE:
The average of temperatures at noontime from Monday to Friday is 50; the lowest one is 45,
what is the possible maximum range of the temperatures?
A. 20
B. 25
C. 40
D. 45
E. 75
9. PROBABILITY OF INTEGER BEING DIVISIBLE BY 8:
If n is an integer from 1 to 96 (inclusive), what is the probability for n*(n+1)*(n+2) being
divisible by 8?
A. 25%
B 50%
C 62.5%
D. 72.5%
E. 75%
10. SUM OF INTEGERS:
If the sum of five consecutive positive integers is A, then the sum of the next five consecutive
integers in terms of A is:
A. A+1 inquiry

B. A+5
C A+25
D 2A
E. 5A
TOUGH AND TRICKY QUESTIONS
1. Word problem / Mixture Problem
A container has 3L of pure wine. 1L from the container is taken out and 2L water is added.The
process is repeated several times. After 19 such operations, qty of wine in mixture is
A. 2/7 L
B. 3/7 L
C. 6/19L
First operation: 3L-1L=2=6/3L of wine left, total 4L;
#2: 6/3L-(6/3)/4=6/3-6/12=18/12=6/4L of wine left, total 5L;
#3: 6/4L-(6/4)/5=6/4-6/20=24/20=6/5L, total 6L;
#4: 6/5L-(6/5)/6=6/5-6/30=30/30=6/6L, total 7L;
....
At this point it's already possible to see the pattern: x=6/(n+2)
n=19 --> x=6/(19+2)=6/21=2/7L
Answer: A.
Discussed at:

http://gmatclub.com/forum/ratio-and-proportion-86312.html

2. Algebra / Modulus
What is the sum of all roots of the equation

Solve for

-->

OR

out. Solving

-->

, BUT as absolute value never negative thus -2 is


or

-->

Answer: -8.
Discussed at:

http://gmatclub.com/forum/sum-of-all-roots-of-the-equation-85988.html#p645659

3. Algebra
If f(x)=5x^2 and g(x)=x^2 + 12x + 85, what is the sum of all values for k such that f (k+2)=g(2k)?
-->

Viete's formula for the roots

AND

and

of equation

So in our case the roots

Answer: 4.
Discussed at:

http://gmatclub.com/forum/quadratic-querie-85989.html#p644602

4. Word problem
On a race track a maximum of 5 horses can race together at a time. There are a total of 25 horses.
There is no way of timing the races. What is the minimum number of races we need to conduct to
get the top 3 fastest horses?
A. 5
B. 7
C. 8
D. 10
E. 11
First 5 races: all horses by five. We'll have the five winners.
Race 6: the winners of previous five races. We'll have the 3 winners.
Now it's obvious that #1 here is the fastest one (gold medal).
For the silver and bronze we'll have 5 pretenders:
1. #2 from the last sixth race,
2. #3 from the last sixth race,
3. the second one from the race with the Gold medal winner from the first five races,
4. the third one from the race with the Gold medal winner from the first five races,
5. the second one from the race with the one which took the silver in the sixth race
Race 7: these five horse: first and second in this one will have the silver and bronze among all 25.
Answer B.
Discussed at:

http://gmatclub.com/forum/good-q-85080.html

5. Coordinate Geometry
In the rectangular coordinate system, points (4, 0) and ( 4, 0) both lie on circle C. What is the
maximum possible value of the radius of C ?
A. 2
B. 4
C. 8
D. 16
E. None of the above
The only thing we can conclude from the question that center lies on the Y-axis. But it could be ANY
point on it, hence we can not determine maximum value of r.
Answer: E.
Discussed at:

http://gmatclub.com/forum/maximum-value-of-the-radius-86703.html

6. Word Problem
In an ensemble of gongs, all gongs have a diameter of either ten inches, or twelve inches or fifteen
inches. In the collection there are 18 ten inch gongs. Half of the gongs in the collection are Tiger
gongs. Of the Tiger gongs, there are equal numbers of ten inch, twelve inch and fifteen inch gongs.
Half of the twelve inch gongs are not Tiger gongs, and half of all gongs are fifteen inches in
diameter. How many gongs are there in the collection?

A. 18
B. 54
C. 72
D. 90
E. 108
If not the wording question won't be hard:
Let x and y be 12 and 15 inches gongs respectively. We know that ten inches are 18.
1.

. We want to calculate

2. "Half of the gongs in the collection are Tiger gongs" -->

3. "Half of the twelve inch gongs are not Tiger gongs" --> means another half IS Tiger gongs, so x/2 is in
Tiger gongs. As "Of the Tiger gongs, there are equal numbers of ten inch, twelve inch and fifteen inch
gongs". --> x/2+x/2+x/2=t -->

4. "Half of all gongs are fifteen inches in diameter" -->

Four unknowns, four equations.


(3)

and (2)

(4)

(1)

-->

-->

-->

-->

Answer: E.
Discussed at:

http://gmatclub.com/forum/set-question-86769.html

7. Word Problems / Fractions


The rate of a certain chemical reaction is directly proportional to the square of the concentration
of chemical A present and inversely proportional to the concentration of chemical B present. If the
concentration of chemical B is increased by 100%, which of the following is closest to the percent
change in the concentration of chemical A required to keep the reaction rate unchanged?
A. 100% decrease
B. 50% decrease
C. 40% decrease
D. 40% increase
E. 50% increase
NOTE: Put directly proportional in nominator and inversely proportional in denominator.
, (well as it's not the exact fraction it should be multiplied by some constant but we

can ignore this in our case).


We are told that B increased by 100%, hence in denominator we have 2B. We want the rate to be the
same. As rate is directly proportional to the SQUARE of A, A should also increase (nominator) by x
percent and increase of A in square should be 2. Which means x^2=2, x=~1.41, which is approximately
40% increase.

Answer: D.
Discussed at:

http://gmatclub.com/forum/ps-gmatprep-challenge-86954.html

8. Rate problem
Car B begins moving at 2 mph around a circular track with a radius of 10 miles. Ten hours later, Car
A leaves from the same point in the opposite direction, traveling at 3 mph. For how many hours
will Car B have been traveling when car A has passed and moved 12 miles beyond Car B?
A. 4pi 1.6
B. 4pi + 8.4
C. 4pi + 10.4
D. 2pi 1.6
E. 2pi 0.8
It's possible to write the whole formula right away but I think it would be better to go step by step:
B speed:

mph;

A speed:

mph (travelling in the opposite direction);

Track distance:

What distance will cover B in 10h:

miles

Distance between B and A by the time, A starts to travel:

Time needed for A and B to meet distance between them divided by the relative
speed:
, as they are travelling in opposite directions relative

speed would be the sum of their rates;


Time needed for A to be 12 miles ahead of B:

So we have three period of times:


Time before A started travelling:

hours;

Time for A and B to meet:

hours;

Time needed for A to be 12 miles ahead of B:

Total time:

hours;

hours.

Answer: B.
Discussed at:

http://gmatclub.com/forum/rates-on-a-circular-track-86675.html

9. Rate problem
A man cycling along the road noticed that every 12 minutes a bus overtakes him and every 4
minutes he meets an oncoming bus. If all buses and the cyclist move at a constant speed, what is
the time interval between consecutive buses?
A. 5 minutes
B. 6 minutes
C. 8 minutes
D. 9 minutes
E. 10 minutes
Let's say the distance between the buses is . We want to determine

, where

speed of bus.
Let the speed of cyclist be .
Every 12 minutes a bus overtakes cyclist:

Every 4 minutes cyclist meets an oncoming bus:

, -->

, -->

Answer: B.
Discussed at:

http://gmatclub.com/forum/bus-86404.html

10. Modulus / Inequality


If
which of the following must be true about

A.
B.

is the

C.
D.
E.

First of all let's solve this inequality step by step and see what is the solution for it, or in other words
let's see in which ranges this inequality holds true.
Two cases for

A.

-->

B.

-->

-->

-->

-->

-->

-->

So given inequality holds true in the ranges:

and

. Which means that

can take

values only from these ranges.


------{-1}xxxx{0}----{1}xxxxxx
Now, we are asked which of the following must be true about
because

can be from the range

Only option which is ALWAYS true is B. ANY


more the

, eg

. Option A can not be ALWAYS true

and

from the ranges

and

will definitely be

, all "red", possible x-es are to the right of -1, which means that all possible x-es are

more than -1.


Answer: B.
Discussed at:

http://gmatclub.com/forum/inequality-68886-20.html

11. Number Properties


A symmetric number of an another one is a number where the digit are reversed. for instance 123
is the symmetric of one of 321. Thus the different of a number and its symmetrical must be
divisible by which of the following?
A. 4
B. 5
C. 6
D. 7
E. 9
Let's consider the example of three digit symmetric numbers {abc} and {cba}. Three digit number can
be represented as: {abc}=100a+10b+c and {cba}=100c+10b+a. The difference would be:

{abc}-{cba}=100a+10b+c-(100c+10b+a)=99a-99c=99(a-c).
Two digit: {ab} and {ba}. {ab}-{ba}=10a+b-(10b+a)=9a-9b=9(a-b)
Hence the difference of two symmetric numbers (2 digit, 3 digit, ...) will always be divisible by 9.
Answer: E.
Discussed at:

http://gmatclub.com/forum/number-properties-hard-question-helpp-89340.html

12. Number properties / Fractions


If 10*x/(x+y)+ 20*y/(x+y)=k and if x is less than y, which of the following could be the value of k?
A. 10
B. 12
C. 15
D. 18
E. 30

Finally we get:

We know that

Hence

is more than

and less than

So,

Only answer between

and

is

There can be another approach:


We have:

, if you look at this equation you'll notice that it's a weighted average.

There are

red boxes and

blue boxes. Red box weight is 10kg and blue box weight 20kg, what is

the average weight of x red boxes and y blue boxes?


k represents the weighted average. As y>x, then the weighted average k, must be closer to 20 than to
10. 18 is the only choice satisfying this condition.
Answer: D (18).
Discussed at:

http://gmatclub.com/forum/must-be-an-easier-way-88620.html#p668499

13. Remainders
What is the remainder when (1!)^3+ (2!)^3 + (3!)^3 +.....(1152!)^3 is divided by 1152?
A. 125
B. 225
C. 325
We have the sum of many numbers:

and want to determine the

remainder when this sum is divided by 1152.


First we should do the prime factorization of 1152:

Consider the third and fourth terms:


not divisible by 1152;
divisible by 1152, and all the other terms after will
be divisible by 1152.
We'll get

and this sum divided by

1152 will result remainder of 225.


Answer: A.
Discussed at:

http://gmatclub.com/forum/numbers-86324.html

14. Number Properties


If X is a positive integer and 405^4 is a multiple of 3^X, what is the largest possible value of X?
A. 5.
B. 12.
C. 16.
D. 20.
E. 26
Given:
is 16 (for

Answer: B.

-->
)

. Hence the largest possible value of x

Discussed at:

http://gmatclub.com/forum/a-is-a-prime-number-a-91113.html

15. Geometry
A square wooden plaque has a square brass inlay in the center ,leaving a wooden strip of uniform
width around the brass square.if the ratio of the brass area to the wooden area is 25 to 39,which
of the following could be the width ,in inches ,of the wooden strip.
I. 1
II. 3
III. 4
A. I only
B. II only
C. III only
D. I and III only
E. I,II and III
Why would ANY width of the strip be impossible?
Let the the side of small square be

Given:

-->

We are asked which value of

Well, expression

and the big square

-->

is possible.

can take ANY value depending on

: 1, 3, 4, 444, 67556, 0,9, ... ANY. Basically

we are given the ratios of the sides (5/8), half of their difference can be any value we choose, there
won't be any "impossible" values at all.

http://gmatclub.com/forum/hard-problem-og-quant-2nd-edition89215.html#p674564
Discussed at:

16. Statistics
If the mean of set S does not exceed mean of any subset of set S , which of the following must be
true about set S ?
I. Set S contains only one element
II. All elements in set S are equal
III. The median of set S equals the mean of set S
A. none of the three qualities is necessary
B. II only
C. III only
D. II and III only
E. I, II, and III
"The mean of set S does not exceed mean of any subset of set S" --> set S can be:
A.
- S contains only one element (eg {7});

B.

- S contains more than one element and all elements are equal (eg{7,7,7,7}).

Why is that? Because if set S contains two (or more) different elements, then we can always consider
the subset with smallest number and the mean of this subset (mean of subset=smallest number) will be
less than mean of entire set (mean of full set>smallest number).
Example: S={3, 5} --> mean of S=4. Pick subset with smallest number s'={3} --> mean of s'=3 --> 3<4.
Now let's consider the statements:
I. Set S contains only one element - not always true, we can have scenario B too (

);

II. All elements in set S are equal - true for both A and B scenarios, hence always true;
III. The median of set S equals the mean of set S - - true for both A and B scenarios, hence always true.
So statements II and III are always true.
Answer: D.
Discussed at:

http://gmatclub.com/forum/ps-challenge-93565.html

17. Number Problems


For every positive even integer n, the function h(n) is defined to be the product of all the even
integers from2 to n, inclusive. If p is the smallest factor of h(100) + 1, then p is
A. between 2 and 10
B. between 20 and 10
C. between 20 and 30
D. between 30 and 40
E. greater than 40

Now, two numbers

and

are consecutive integers. Two

consecutive integers are co-prime, which means that they don't share ANY common factor but 1. For
example 20 and 21 are consecutive integers, thus only common factor they share is 1.
As
above

has all numbers from 1 to 50 as its factors, according to


won't have ANY factor from 1 to 50. Hence

), the smallest

factor of

will be more than 50.

Answer: E.

http://gmatclub.com/forum/hard-question-function-and-factors90863.html#p693396
Discussed at:

18. Rate Problem


A bus from city M is travelling to city N at a constant speed while another bus is making the same
journey in the opposite direction at the same constant speed. They meet in point P after driving
for 2 hours. The following day the buses do the return trip at the same constant speed. One bus is
delayed 24 minutes and the other leaves 36 minutes earlier. If they meet 24 miles from point P,
what is the distance between the two cities?
A. 48
B. 72
C. 96
D. 120
E. 192
Distance between the cities .

First meeting point , as both buses travel at the same constant speed and leave the cities same time

they meet at the halfway.


Total time to cover the

4 hours, as the buses meet in 2 hours.

On the second day first bus traveled alone 1 hour (36min +24min), hence covered

, and

left cover.
They meet again at the halfway of

, which is 24 miles from :

Answer: E.
Discussed at:

http://gmatclub.com/forum/two-buses-same-speed-head-spinning-86478.html

19. Number Properties


If
is multiple of , and
multiple of

where

and

are prime, which of the following must be a

is

A
B.
C.
D.
E.

and

,(

and

are primes).

Q:

Well obviously either

or

is . As we are asked to determine which choice MUST be multiple of

right answer choice must have BOTH,


by

and

in power of 2 or higher to guarantee the divisibility

. Only D offers this.

Answer: D.
Discussed at:

http://gmatclub.com/forum/gmatprep-ps-questions-need-help-93922.html

20. Statistics
If d is the standard deviation x, y, and z, what is the standard deviation of x+5, y+5, z+5
A. d
B. 3d
C. 15d
D. d+5
E. d+15
TIP:
If we add or subtract a constant to each term in a set:
Mean will increase or decrease by the same constant.
SD will not change.
If we increase or decrease each term in a set by the same percent (multiply by a constant):
Mean will increase or decrease by the same percent.
SD will increase or decrease by the same percent.
So in our case SD won't change as we are adding 5 to each term in a set --> SD=d.
Answer: A.
Discussed at:

http://gmatclub.com/forum/gmatprep-ps-questions-need-help-93922.html

21. Work Problem


It takes machine A 'x' hours to manufacture a deck of cards that machine B can manufacture in 'y'
hours. If machine A operates alone for 'z' hours and is then joined by machine B until 100 decks are
finished, for how long will the two machines operate simultaneously?
A. (100xy z)/(x + y)

B. y(100x z)/(x + y)
C. 100y(x z)/(x + y)
D. (x + y)/(100xy z)
E. (x + y z)/100xy
Note that we are asked: "for how long will the two machines operate simultaneously?".
In first

hours machine A alone will manufacture

decks. So there are

decks left

to manufacture. Combined rate of machines A and B would be

decks/hour, (remember

we can easily sum the rates).


As

, then

Answer: B.
Discussed at:

http://gmatclub.com/forum/deck-of-cards-95608.html

22. Fractions
Every day a certain bank calculates its average daily deposit for that calendar month up to and
including that day. If on a randomly chosen day in June the sum of all deposits up to and including
that day is a prime integer greater than 100, what is the probability that the average daily deposit
up to and including that day contains fewer than 5 decimal places?
A. 1/10
B. 2/15
C. 4/15
D. 3/10
E. 11/30
Theory:
Reduced fraction

(meaning that fraction is already reduced to its lowest term) can be expressed as

terminating decimal if and only


negative integers. For example:

to

. Fraction

(denominator) is of the form


is a terminating decimal

is also a terminating decimal, as

, where
, as

and

are non-

(denominator) equals

and denominator

Note that if denominator already has only 2-s and/or 5-s then it doesn't matter whether the
fraction is reduced or not.
For example

, (where x, n and m are integers) will always be terminating decimal.

(We need reducing in case when we have the prime in denominator other then 2 or 5 to see whether it
could be reduced. For example fraction
has 3 as prime in denominator and we need to know if it

can be reduced.)

BACK TOT THE ORIGINAL QUESTION:


Question: does

has less than 5 decimal places? Where

and

is the

chosen day.
If the chosen day, , is NOT of a type
then

(where

and

are nonnegative integers)

will not be a terminating decimal and thus will have more than 5 decimal places.

How many such days are there of a type


,

, ,

: 1, 2, 4, 5, 8, 10, 16, 20, 25 (

), total of 9 such days (1st of

June, 4th of June, ...).


Now, does

divided by any of these

as

have fewer than 5 decimal places? Yes,

for any such

(10,000 is divisible by all these numbers: 1, 2, 4, 5, 8, 10,

16, 20, 25).


So, there are 9 such days out of 30 in June:

Answer: D.
Discussed at:

http://gmatclub.com/forum/mgmat-challenge-decimals-on-deposit-97456.html

23. Sequences
What is the sixtieth term in the following sequence? 1, 2, 4, 7, 11, 16, 22...
A. 1,671
B. 1,760
C. 1,761
D. 1,771
E. 1,821
First we should find out what is the pattern of this sequence. Write the terms of the sequence as
below:
;
;
;
;

;
;
;
...
So the

term of this sequence equals to first term plus the sum of first

integers.

Answer: D.
Discussed at:

http://gmatclub.com/forum/help-tough-problem-on-exponential-sequence-

97907.html
24. Powers / Number Properties
If 3^(6x) = 8,100, what is the value of [3^(x 1)]^3 ?
A. 90
B. 30
C. 10
D. 10/3
E. 10/9
If

, what is the value of

? (It's not hard at all).

-->

Answer: D.

http://gmatclub.com/forum/anything-wrong-in-this-problem-can-anyone-dare-tosolve-98777.html
Discussed at:

25. Inequalities / Algebra


If x and y are integers such that (x+1)^2 less than equal to 36 and (y-1)^2 less than 64. What is the
largest possible and minimum possible value of xy.
-->

-->

-->

-->

-->

-->

, as

is an integer we can

rewrite this inequality as

We should try extreme values of

and

to obtain min and max values of

Min possible value of

is for

and

Max possible value of

is for

and

-->

;
-->

Solving with absolute values gives the same results:


means

-->

means

Discussed at:

-->

http://gmatclub.com/forum/in-equalities-how-to-handle-an-expression-with-squares-

97992.html
26. Powers
The operation x#n for all positive integers greater than 1 is defined in the following manner:
x#n = x to the power of x#(n-1)
If x#1 = x, which of the following expressions has the greatest value?
A. (3#2)#2
B. 3#(1#3)
C. (2#3)#2
D. 2#(2#3)
E. (2#2)#3
Couple of things before solving:
If exponentiation is indicated by stacked symbols, the rule is to work from the top down, thus:
and not
, which on the other hand equals to
.

So:

and not

Back to the original question:


Let's replace # by @ as # looks like the symbol "not equal to" and it might confuse someone.
and

, as

;
;
;

...
Basically n in x@n represents the # of stacked x-es.
A.

B.

C.

D.

this will be huge number

2^2^2^2^2^2^2^2^2^2^2^2^2^2^2^2=2^2^2^2^2^2^2^2^2^2^2^2^2^2^4=2^2^2^2^2^2^2^2^2^2^2^2^2^1
6=.... ;
E.

2^2^2^2^2^2^2^2^2^2^2^2^2^2^2^2 will be much bigger number than numbers from other answer
choices.
Answer: D.
Discussed at:

http://gmatclub.com/forum/just-800-level-question-99064.html

27. Overlapping Sets


3/8 of all students at Social High are in all three of the following clubs: Albanian, Bardic, and
Checkmate. 1/2 of all students are in Albanian, 5/8 are in Bardic, and 3/4 are in Checkmate. If
every student is in at least one club, what fraction of the student body is in exactly 2 clubs?
A. 1/8
B. 1/4
C. 3/8
D. 1/2
E. 5/8
Let's # of students at Social High be 8 (I picked 8 as in this case 3/8 of total and 5/8 of total will be an
integer).
3/8 of all students at Social High are in all three clubs --> 3/8*8=3 people are in exactly 3 clubs;
1/2 of all students are in Albanian club --> 1/2*8=4 people are in Albanian club;
5/8 of all students are in Bardic club --> 5/8*8=5 people are in Bardic club;
3/4 of all students are in Checkmate club --> 3/4*8=6 people are in Checkmate club;
Also as every student is in at least one club then # of students in neither of clubs is 0;

Total=A+B+C-{# of students in exactly 2 clubs}-2*{# of students in exactly 3 clubs}+{# of students


in neither of clubs};
8=4+5+6-{# of students in exactly 2 clubs}-2*3+0 --> {# of students in exactly 2 clubs}=1, so
fraction is 1/8.
Answer: A.
Discussed at:

http://gmatclub.com/forum/the-quest-for-700-weekly-gmat-challenge-99549.html

28. Word Problem


A certain city with population of 132,000 is to be divided into 11 voting districts, and no district is
to have a population that is more than 10 percent greater than the population of any other
district. What is the minimum possible population that the least populated district could have?
A. 10,700
B. 10,800
C. 10,900
D. 11,000
E. 11,100
As "no district is to have a population that is more than 10 percent greater than the population
of any other district", then the districts can have only two population #:
and
.

So we want to minimize

. To minimize

we should make only one district to have that # of

population (minimum possible) and the rest 10 districts to have

# of population (maximum

possible).
-->

-->

Answer: D.
Discussed at:

http://gmatclub.com/forum/gmat-prep-ps-93369.html

29. Remainders
What is the remainder when (18^22)^10 is divided by 7 ?
1
B2
C3
D4
E5
I think this question is beyond the GMAT scope. It can be solved with Fermat's little theorem, which is
not tested on GMAT. Or another way:
now if we expand this all terms but the last one will have 14 as
multiple and thus will be divisible by 7. The last term will be
when

is divided by 7.

. So we should find the remainder

2^1 divided by 7 yields remainder of 2;


2^2 divided by 7 yields remainder of 4;
2^3 divided by 7 yields remainder of 1;
2^4 divided by 7 yields remainder of 2;
2^5 divided by 7 yields remainder of 4;
2^6 divided by 7 yields remainder of 1;
...
So the remainder repeats the pattern of 3: 2-4-1. So the remainder of
same as

divided by 7 (440=146*3+2).

divided by 7 would be the

divided by 7 yields remainder of 4.

Answer: D.
Discussed at:

http://gmatclub.com/forum/remainder-99724.html

30. Number Properties


For any positive integer n, the length of n is defined as the number of prime factors whose product
is n. For example; the length of 75 is 3, since 75= 3*5*5. How many two digit positive integers
have the length 6?
A. None
B. One
C. Two
D. Three
E. Four
Basically the length of the integer is the sum of the powers of its prime factors.
Length of six means that the sum of the powers of primes of the integer (two digit) must be . First we
can conclude that
six that has

can not be a factor of this integer as the smallest integer with the length of

as prime factor is

(length=5+1=6), not a two digit integer.

The above means that the primes of the two digit integers we are looking for can be only
.

max value of

Let's start with the highest value of

and

is .

(length=6+0=6);
(length=5+1=6);

(length=4+2=6) not good as 144 is a three digit integer.

and/or

With this approach we see that actually

Answer: B.
Discussed at:

http://gmatclub.com/forum/700-algrbra-need-help-again-thanks-so-much-

90320.html
31. Statistics
E is a collection of four ODD integers and the greatest difference between any two integers in E is
4. The standard deviation of E must be one of how many numbers?
A3
B4
C5
D6
E7
Let the smallest odd integer be 1, thus the largest one will be 5. We can have following 6 types of sets:
1.
2.
3.
4.
5.
6.

{1,
{1,
{1,
{1,
{1,
{1,

1,
1,
1,
3,
3,
5,

1,
3,
5,
3,
5,
5,

5}
5}
5}
5}
5}
5}

--> mean=2 --> |mean-x|=(1, 1, 1, 3);


--> mean=2.5 --> |mean-x|=(1.5, 1.5, 0.5, 2.5);
--> mean=3 --> |mean-x|=(2, 2, 2, 2);
--> mean=3 --> |mean-x|=(2, 0, 0, 2);
--> mean=3.5 --> |mean-x|=(2.5, 0.5, 1.5, 1.5);
--> mean=4 --> |mean-x|=(3, 1, 1, 1).

CALCULATING STANDARD DEVIATION OF A SET {x1, x2, ... xn}:


1. Find the mean,
, of the values.
2. For each value

calculate its deviation (

) from the mean.

3. Calculate the squares of these deviations.


4. Find the mean of the squared deviations. This quantity is the variance.
5. Take the square root of the variance. The quantity is th SD.
Expressed by formula:

You can see that deviation from the mean for 2 pairs of the set is the same, which means that SD of 1
and 6 will be the same and SD of 2 and 5 also will be the same. So SD of such set can take only 4
values.
Answer: B.
Discussed at:

http://gmatclub.com/forum/hard-standard-deviation-99774.html

32. Geometry
The dimensions of a rectangular solid are 4 inches, 5 inches, and 8 inches. If a cube, a side of
which is equal to one of the dimensions of the rectangular solid, is placed entirely within
the sphere just large enough to hold the cube, what the ratio of the volume of the cube to the
volume within the sphere that is not occupied by the cube?
A. 10:17
B. 2:5
C. 5:16

D. 25:7
E. 32:25
The cube must have a side of 4 inches to fit in rectangular solid. Now as the cube is inscribed in sphere
then the radius of this sphere equals to half of the cube's diagonal -->
, so
radius of the sphere is

Volume of the cube will be

Volume of the sphere will be

The ratio of the volume of the cube to the volume within the sphere that is not occupied by the cube
is
, and after some calculations you'll get that this ratio is closer to 10/17 than to any other

ratios in answer choices.


Answer: A.
P.S. I really doubt that GMAT would offer such a problem, as it requires lengthy calculations and bad
approximations ( ,
).

Discussed at:

http://gmatclub.com/forum/hard-geometry-99533.html

33. Coordinate Geometry


On the coordinate plane (6, 2) and (0, 6) are the endpoints of the diagonal of a square. What is the
distance between point (0, 0) and the closest vertex of the square?
A. 1/sqrt (2)
B. 1
C. sqrt (2)
D. sqrt (3)
E. 2*sqrt (3)
Given endpoints of diagonal of a square: B(0,6) and D(6,2). Let other vertices be A (closest to the
origin, left bottom vertex) and C (farthest to the origin).
Length of the diagonal would be:

Coordinates of the midpoint M of the diagonal would be:

Slope of the line segment AM*Slope of the line segment BD=-1 (as they are perpendicular to each other)
-->
-->
-->

Distance between the unknown vertices to the midpoint is half the diagonal:
-->
-->

or

-->

-->

or

Hence the coordinates of the point A(1,1) and point C (5,7). Closest to the origin is A.
Distance

Answer: C.
Discussed at:

http://gmatclub.com/forum/coordinate-plane-90772.html

34. Word Problem


Eight litres are drawn off from a vessel full of water and substituted by pure milk. Again eight litres
of the mixture are drawn off and substituted by pure milk. If the vessel now contains water and
milk in the ratio 9:40, find the capacity of the vessel.
A. 21 litres
B. 22 litres
C. 20 litres
D. 14 litres
E. 28 litres
Let the capacity of the vessel be

After the first removal there would be


the water would be

liters of water left in the vessel. Note that the share of

After the second removal, the removed mixture of 8 liters will contain
will be

liters of water left.

As the ratio of water to milk after that is

would be

liters of water, so there

, then the ratio of water to the capacity of the vessel

So

-->

-->

-->

Answer: D.

http://gmatclub.com/forum/eight-litres-are-drawn-off-from-a-vessel-full-of-waterand-s-95749.html
Discussed at:

35. Inequality / Algebra


If
, which of the following statements could be true?
1.
2.
3.

A..) 1 only
B. 1 & 2 only
C. 1 & 3 only
D. 2 & 3 only
E. 1,2 & 3
As this is a COULD be true question then even one set of numbers proving that statement holds true is
enough to say that this statement should be part of correct answer choice.
Given:

1.

--> the easiest one: if

well as given statement

2.

and

and

--> this set satisfies

), so let's try fractions: if

then again the stem and this statement hold true. So 2 also COULD be true.

--> let's make

some big number, let's say 1,000. Next, let's try the fractions for

for the same reason as above (reverse order of

and ):

statement hold true for this set of numbers. So 3 also COULD be true.
Answer: E.
Discussed at:

as

. So 1 COULD be true.

--> we have reverse order than in stem (

3.
and

http://gmatclub.com/forum/x-y-2-z-100465.html

and

. The stem and this

36. Sequences
Series
is such that

for any pair of positive integers

. If

is a

positive integer, which of the following is possible?


I.

II.

is the only integer in the series

III. The series does not contain negative numbers


I only
II only
I and III only
II and III only
I, II, and III
A set of numbers

, ... have the following properties:

and

so

We should determine whether the options given below can occur (note that the question is which can
be true, not must be true).
I.

--> as

reduce by

II.

, then

-->

which is not true. Hence this option cannot be true.

is the only integer in the series. If

-->

-->

, then all other terms will be non-integers


,

, and so on. Hence this option can be

true.
III. The series does not contain negative numbers --> as given
that
,
then

Answer: D (II and III only).

-->

, hence this option is always true.

Discussed at:

http://gmatclub.com/forum/data-suffiency-tough-question-96538.html

37. Number Properties


If a and b are positive integers such that a/b = 2.86, which of the following must be a divisor of a?
A.1. 10
B. 13
C. 18
D. 26
E. 50
-->

143 (50 has none of them). Hence

, for

to be an integer

must have all the factors of

must be divisible by both 11 and 13.

Answer: B.
Discussed at:

http://gmatclub.com/forum/prime-factor-100704.html

38. Rate problem


A boat travelled upstream 90 miles at an average speed of (v-3) miles per hour and then traveled
the same distance downstream at an average speed of (v+3) miles per hour. If the trip upstream
took a half hour longer than the trip downstream, then how many hours did it take the boat to
travel downstream?
A. 2.5
B. 2.4
C. 2.3
D. 2.2
E. 2.1
Trip upstream took
hours and trip downstream took
hours. Also given that the difference in

times was

hours -->

-->

>

-->

-->

Trip downstream took

-->

-->

hours.

Answer: A.
Discussed at:
39. Modulus

http://gmatclub.com/forum/difficult-ps-problem-help-100767.html

--

If x<0, then

equals:

A.
B.
C.
D.
E.
Remember:

The point here is that square root function can not give negative result: wich means
that
.

So

. But what does

equal to?

Let's consider following examples:


If
-->
If

-->

So we got that:
, if

, if

What function does exactly the same thing? The absolute value function! That is why

Back to the original question:

Or just substitute the value let


>

-.

Answer: A.
Discussed at:

http://gmatclub.com/forum/square-root-and-modulus-100303.html

41. Word Problem

A certain car averages 25 miles per gallon when driving in the city and 40 miles to the gallon when
driving on the highway. According to these rates, which of the following is closest to the number of
miles per gallon that the car averages when it is driven 10 miles in the city and 50 miles on the
highway?
A. 28
B. 30
C. 33
D. 36
E. 38
Car averages x miles per gallon, means that 1 gallon is enough to drive x miles.
We are asked to find average miles per gallon (miles/gallon) --> average miles per gallon would
be total miles driven divided by total gallons used(miles/gallon).
Total miles driven is 10+50=60.
As car averages 25 miles per gallon in the city for 10 miles in the city it will use 10/25=0.4 gallons;
As car averages 40 miles per gallon on the highway for 50 miles on the highway it will use 50/40=1.25
gallons;
So average miles per gallon equals to

Answer: D.
Discussed at:

http://gmatclub.com/forum/more-prep-questions-98942.html

42. Remainders
When 26854 and 27584 are divided by a certain two digit prime number, the remainder obtained is
47. Which of the following choices is a possible value of the divisor?
A. 61
B. 71
C. 73
D. 89
Given:

Where

is the prime number (divisor) and

and

are the quotients (integers

Subtract equations:
Product of two integers

-->
, two digit prime numberand

digit prime number, it can be only

.
is

(2 and 5 are single digit).

Answer: 73.
Discussed at:

).

http://gmatclub.com/forum/number-system-60282.html

, as

is two

43. Remainders
If x and y are positive integers and x/y has a remainder of 5, what is the smallest possible value of
xy?
Given

, where

when

is a quotient, an integer

, in that case

. Which means that the least value of

. This basically means that numerator

is

, is less than denominator

.
Now the smallest denominator
and

-->

Discussed at:

, which is more than numerator

is . so we have

http://gmatclub.com/forum/remainder-of-89470.html

44. Remainders
When positive integer n is divided by 5, the remainder is 1. When n is divided by 7, the remainder
is 3. What is the smallest positive integer k such that k+n is a multiple of 35?
A. 3
B. 4
C. 12
D. 32
E. 35
Positive integer n is divided by 5, the remainder is 1 -->

, where

is the quotient --> 1, 6,

11, 16, 21, 26, 31, ...


Positive integer n is divided by 7, the remainder is 3 -->

, where

is the quotient --> 3,

10, 17, 24, 31, ....


You can not use the same variable for quotients in both formulas, because quotient may not be the
same upon division n by two different numbers.
For example 31/5, quotient q=6 but 31/7, quotient p=4.
There is a way to derive general formula for

(of a type

, where

is divisor and

is a remainder) based on above two statements:


Divisor

would be the least common multiple of above two divisors 5 and 7, hence

Remainder

would be the first common integer in above two patterns, hence

Therefore general formula based on both statements is


integer k such that k+n is a multiple of 35 is 4 -->

Answer: B.

. Thus the smallest positive


.

Discussed at:

http://gmatclub.com/forum/good-problem-90442.html

45. Statistics
A certain list has an average of 6 and a standard deviation of d (d is positive). Which of the
following pairs of data when added to the list, must result in a list of 102 data with standard
deviation less than d?
A. (-6;0)
B. (0;0)
C. (0;6)
D. (0;12)
E. (6;6)
"Standard deviation shows how much variation there is from the mean. A low standard deviation
indicates that the data points tend to be very close to the mean, whereas high standard deviation
indicates that the data are spread out over a large range of values."
So when we add numbers, which are far from the mean we are stretching the set making SD bigger and
when we add numbers which are close to the mean we are shrinking the set making SD smaller.
According to the above adding two numbers which are closest to the mean will shrink the set most,
thus decreasing SD by the greatest amount.
Closest to the mean are 6 and 6 (actually these numbers equal to the mean) thus adding them will
definitely shrink the set, thus decreasing SD.
Answer: E.
Discussed at:

http://gmatclub.com/forum/standard-deviation-97473.html

46. Rate Problem


If a motorist had driven 1 hour longer on a certain day and at an average rate of 5 miles per hour
faster, he would have covered 70 more miles than he actually did. How many more miles would he
have covered than he actually did if he had driven 2 hours longer and at an average rate of 10
miles per hour faster on that day?
A. 100
B. 120
C. 140
D. 150
E. 160
Let

be the actual time and

be the actual rate.

"If a motorist had driven 1 hour longer on a certain day and at an average rate of 5 miles per hour
faster, he would have covered 70 more miles than he actually did" -->
->

-->

"How many more miles would he have covered than he actually did if he had driven 2 hours longer and
at an average rate of 10 miles per hour faster on that day?" -->
->

-->

--> as from above

then

--> so

Answer: D.
OR another way:
70 miles of surplus in distance is composed of driving at 5 miles per hour faster for
hours plus driving for
>

miles per hour for additional 1 hour -->

--

With the same logic, surplus in distance generated by driving at 10 miles per hour faster for 2 hours
longer will be composed of driving at 10 miles per hour faster for hours plus driving for
miles per hour for additional 2 hour -->
>

--> as from above

-, then

Answer: D.
Discussed at:

http://gmatclub.com/forum/ps-motorist-61729.html

47. Fractions
The ratio of two positive numbers is 3 to 4. If k is added to each number the new ratio will be 4 to
5, and the sum of the numbers will be 117. What is the value of k?
A. 1
B. 13
C. 14
D. 18
E. 21
The ratio of two positive numbers is 3 to 4 -->

, for some positive integer

If k is added to each number the new ratio will be 4 to 5 -->

>

-->

--

The sum of the numbers will be 117 (I believe it means that the sum of the numbers after we add k to
each) -->
-->
--> as from above
-->
-->

Answer: B.
Discussed at:

http://gmatclub.com/forum/interesting-ratio-problem-95198.html

48. Word Problem / Percent


Before being simplified, the instructions for computing income tax in country R were to add 2
percent of ones annual income to the average (Arithmetic mean) of 100 units of country Rs
currency and 1 percent of ones annual income. Which of the following represents the simplified
formula for computing the income tax, in country Rs currency, for a person in that country whose
annual income is I.
A. 50 + (I/200)
B. 50 + (3I/100)
C. 50 + (I/40)
D. 100 + (I/50)
E. 100 + (3I/100)
Tax is the sum of the following:
2 percent of one's annual income -

The average (arithmetic mean) of 100 units of country R's currency and 1 percent of one's annual
income .

Answer: C.
Discussed at:

http://gmatclub.com/forum/ps-from-gmat-prep-91752.html

49. Rate Problem


Circular gears P and Q start rotating at the same time at constant speeds. Gear P makes 10
revolutions per minute, and gear Q makes 40 revolutions per minute. How many seconds after the
gears start rotating will gear Q have made exactly 6 more revolutions than gear P?
A. 6
B. 8
C. 10
D. 12
E. 15

Note that we are given revolutions per minute and asked about revolutions in seconds. So we should
transform per minute to per second.
Gear P makes 10 revolutions per minute --> gear P makes 10/60 revolutions per second;
Gear Q makes 40 revolutions per minute --> gear Q makes 40/60 revolutions per second.
Let

be the time in seconds needed for Q to make exactly 6 more revolutions than gear P

-->

-->

Answer: D.
Discussed at:

http://gmatclub.com/forum/gmat-prep-question-99043.html

50. If k is an integer, and 35^2-1/k is an integer, then k could be each of the following, EXCEPT
A. 8
B. 9
C. 12
D. 16
E. 17

From the answer choices only

is not a factor of numerator, hence in case

won't be an integer, hence k can not be 16.


Answer: D.
Discussed at:

http://gmatclub.com/forum/35-2-1-k-94473.html

1. General Structure

Intro - Restate argument, point out flaws or state intention to discuss them below
1st Para - First,...
2nd Para - Second/In addition,...
3rd Para - Third/Finally,...
Conclusion - The argument is flawed/weak/unconvincing because of the above
-mentioned...Ultimately, the argument can be strengthened if/by...

2. Structural Word (should be all over the essays)

1.
2.
3.
4.
5.
6.
7.

Supporting examples - for example, to illustrate, for instance, because, specifically


Additional support - furthermore, in addition, similarly, just as, also, as a result,
moreover
Importance - surely, truly, undoubtedly, clearly, in fact, most importantly
Contrast - on the contrary, yet, despite, rather, instead, however, although, while
Decide against - one cannot deny that, it could be argued that, granted, admittedly
Ying-yang - on the one hand/on the other hand
Concluding - therefore, in summary, consequently, hence, in conclusion, ultimately,
in closing

3. Templates

Intro:
The argument claims that ....(restate)
Stated in this way the argument:
a) manipulates facts and conveys a distorted view of the situation
b) reveals examples of leap of faith, poor reasoning and ill-defined terminology
c) fails to mention several key factors, on the basis of which it could be evaluated
The conclusion of the argument relies on assumptions for which there is no clear evidence. Hence, the
argument is weak/unconvincing and has several flaws.
1st Para:
First, the argument readily assumes that......
This statement is a stretch....
For example,...
Clearly,...
The argument could have been much clearer if it explicitly stated that...
2nd Para:
Second, the argument claims that....
This is again a very weak and unsupported claim as the argument does not demonstrate any correlation
between....and...
To illustrate,...
While,...
However,....indeed....

In fact, it is not at all clear...rather....


If the argument had provided evidence that.....then the argument would have been a lot more
convincing.
3rd Para:
Finally,...
(pose some questions for the argument).....Without convincing answers to these questions, one is left
with the impression that the claim is more of a wishful thinking rather than substantive evidence.
Conclusion:
In conclusion, the argument is flawed for the above-mentioned reasons and is therefore unconvincing.
It could be considerably strengthened if the author clearly mentioned all the relevant facts....
In order to assess the merits of a certain situation/decision, it is essential to have full knowledge of all
contributing factors. In this particular case....
Without this information, the argument remains unsubstantiated and open to debate.

Das könnte Ihnen auch gefallen